Binomially Amazing! 4

n = 0 1729 ( n 1729 2 ) 2 ( 1729 n ) ( 1 2 ) 1729 = A B \large \sum _{ n=0 }^{ 1729 }{ { \left( n-\frac { 1729 }{ 2 } \right) }^{ 2 }\left( \begin{matrix} 1729 \\ n \end{matrix} \right) { \left( \frac { 1 }{ 2 } \right) }^{ 1729 }=\frac { A }{ B } }

Find: A + B A+B


The answer is 1733.

This section requires Javascript.
You are seeing this because something didn't load right. We suggest you, (a) try refreshing the page, (b) enabling javascript if it is disabled on your browser and, finally, (c) loading the non-javascript version of this page . We're sorry about the hassle.

2 solutions

Wing Tang
Feb 22, 2016

I'm using both differentiation and binomial theorem here.

Solution : \textbf{Solution}:

Let f ( x ) : = n = 0 1729 ( 1729 n ) x 1729 = ( 1 + x ) 1729 f(x) := \sum^{1729}_{n=0} {1729 \choose n} x^{1729} = (1+x)^{1729} . Clearly, putting x = 1 x=1 into f ( x ) f(x) yields

\begin{aligned} \sum^{1729}_{n=0} {1729 \choose n} = 2^{1729}\tag{0} \end{aligned}

Differentiating f f w.r.t. x x once and twice respectively and putting x = 1 x=1 yield

\begin{aligned} \sum^{1729}_{r=0} r {1729 \choose n} &= 1729 \cdot 2^{1728}\tag{1} \\ \sum^{1729}_{r=0} r(r-1) {1729 \choose n} &= 1729 \cdot 1728 \cdot 2^{1727}\quad \quad \tag{2}\\ \end{aligned}

We can expand the left-hand side of (2): r = 0 1729 r 2 ( 1729 n ) r = 0 1729 r ( 1729 n ) = 1729 1728 2 1727 r = 0 1729 r 2 ( 1729 n ) = r = 0 1729 r ( 1729 n ) + 1729 1728 2 1727 = 1729 2 1728 + 1729 1728 2 1727 (1) is used \begin{aligned} \sum^{1729}_{r=0} r^2 {1729 \choose n} - \sum^{1729}_{r=0} r {1729 \choose n} &= 1729 \cdot 1728 \cdot 2^{1727}\\ \sum^{1729}_{r=0} r^2 {1729 \choose n} &= \sum^{1729}_{r=0} r {1729 \choose n} + 1729 \cdot 1728 \cdot 2^{1727}\\ &= 1729 \cdot 2^{1728} + 1729 \cdot 1728 \cdot 2^{1727}\quad \quad \quad \quad \text{(1) is used} \end{aligned}

Now the LHS of the given equation

n = 0 1729 ( n 1729 2 ) 2 ( 1729 n ) ( 1 2 ) 1729 \begin{aligned}\sum^{1729}_{n=0} \left(n - \frac{1729}{2}\right)^2 {1729 \choose n} \left(\frac{1}{2}\right)^{1729}\end{aligned}

can be broken down into

( 1 2 ) 1729 n = 0 1729 n 2 ( 1729 n ) 1729 ( 1 2 ) 1729 n = 0 1729 n ( 1729 n ) + ( 1729 2 ) 2 ( 1 2 ) 1729 n = 0 1729 ( 1729 n ) \begin{aligned}\left(\frac{1}{2}\right)^{1729} \sum^{1729}_{n=0} n^2 {1729 \choose n} - 1729 \left(\frac{1}{2}\right)^{1729} \sum^{1729}_{n=0} n {1729 \choose n} + \left(\frac{1729}{2}\right)^2 \left(\frac{1}{2}\right)^{1729}\sum^{1729}_{n=0} {1729 \choose n}\end{aligned}

Putting the above results into this equation yields ( 1 2 ) 1729 ( 1729 2 1728 + 1729 1728 2 1727 ) 1729 ( 1 2 ) 1729 1729 2 1728 + ( 1729 2 ) 2 ( 1 2 ) 1729 2 1729 = 1729 2 + 1729 × 1728 4 172 9 2 2 + 172 9 2 4 = 1729 4 \begin{aligned} &\left(\frac{1}{2}\right)^{1729} \left(1729 \cdot 2^{1728} + 1729 \cdot 1728 \cdot 2^{1727} \right)- 1729 \left(\frac{1}{2}\right)^{1729} \cdot 1729 \cdot 2^{1728}+ \left(\frac{1729}{2}\right)^2 \left(\frac{1}{2}\right)^{1729}\cdot 2^{1729}\\ =& \frac{1729}{2} + \frac{1729 \times 1728}{4} - \frac{1729^2}{2} + \frac{1729^2}{4}\\ =&\frac{1729}{4}\end{aligned}

and hence A + B = 1733 A + B = 1733 where we have assumed A A and B B are positive coprime integers. \Box

I Generalised And Got Answer As n/4 . (Here it is 1729)

Prakhar Bindal - 5 years, 3 months ago

Log in to reply

Since there are no direct manipulations of this particular number, I am not surprised that it will be n 4 \frac{n}{4} . Generalising every result is always a good habit.

Wing Tang - 5 years, 3 months ago

Log in to reply

Thanks! . i try as far as i can generalise a result (but sometimes it is very painful becoz of tedious manipulations) :)

Prakhar Bindal - 5 years, 3 months ago

Log in to reply

@Prakhar Bindal Experience can ease that pain.

Wing Tang - 5 years, 3 months ago

Wow! Nice way to use differentiation. Thanks!

Aditya Kumar - 5 years, 3 months ago

Log in to reply

There are lots of nice tricks for binomials alone. Try the problem and read my solution here

Wing Tang - 5 years, 3 months ago
Brian Riccardi
Feb 18, 2016

I will use Binomial Theorem , in particular n = 0 m ( m n ) ( 1 2 ) n ( 1 2 ) m n = ( 1 2 + 1 2 ) m = 1 \sum_{n=0}^{m}{m \choose n}(\frac{1}{2})^{n}(\frac{1}{2})^{m-n}=(\frac{1}{2}+\frac{1}{2})^m=1 \\

And the property k ( n k ) = n ( n 1 k 1 ) k{n \choose k}=n{n-1 \choose k-1}

Let

S = n = 0 1729 ( n 2 1729 n + 172 9 2 4 ) ( 1729 n ) ( 1 2 ) 1729 S=\sum_{n=0}^{1729}(n^2-1729n+\frac{1729^2}{4}){1729 \choose n}(\frac{1}{2})^{1729}

We can split S S in three sums:

S 1 = n = 0 1729 n 2 ( 1729 n ) ( 1 2 ) 1729 S 2 = n = 0 1729 n ( 1729 n ) ( 1 2 ) 1729 S 3 = n = 0 1729 ( 1729 n ) ( 1 2 ) 1729 S = S 1 1729 S 2 + 172 9 2 4 S 3 S_1=\sum_{n=0}^{1729}n^2{1729 \choose n}(\frac{1}{2})^{1729} \\ S_2=\sum_{n=0}^{1729}n{1729 \choose n}(\frac{1}{2})^{1729} \\ S_3=\sum_{n=0}^{1729}{1729 \choose n}(\frac{1}{2})^{1729} \\ S=S_1-1729S_2+\frac{1729^2}{4}S_3

Manipulating S 3 n = 0 1729 ( 1729 n ) ( 1 2 ) n ( 1 2 ) 1729 n = 1 \\ \textbf{Manipulating } S_3 \\ \sum_{n=0}^{1729}{1729 \choose n}(\frac{1}{2})^{n}(\frac{1}{2})^{1729-n}=1

Manipulating S 2 n = 0 1729 n ( 1729 n ) ( 1 2 ) 1729 = 1729 2 n = 0 1729 ( 1728 n 1 ) ( 1 2 ) 1728 = 1729 2 n = 1 1728 ( 1728 n ) ( 1 2 ) 1728 = 1729 2 n = 0 1728 ( 1728 n ) ( 1 2 ) n ( 1 2 ) 1728 n = 1729 2 ( 1 ) = 1729 2 \\ \textbf{Manipulating } S_2 \\ \sum_{n=0}^{1729}n{1729 \choose n}(\frac{1}{2})^{1729}\\ =\frac{1729}{2}\sum_{n=0}^{1729}{1728 \choose {n-1}}(\frac{1}{2})^{1728}\\ =\frac{1729}{2}\sum_{n=-1}^{1728}{1728 \choose {n}}(\frac{1}{2})^{1728}\\ =\frac{1729}{2}\sum_{n=0}^{1728}{1728 \choose {n}}(\frac{1}{2})^{n}(\frac{1}{2})^{1728-n}=\frac{1729}{2}(1)=\frac{1729}{2}

Manipulating S 1 n = 0 1729 n 2 ( 1729 n ) ( 1 2 ) 1729 = 1729 n = 0 1729 ( n 1 + 1 ) ( 1728 n 1 ) ( 1 2 ) 1729 = 1729 [ n = 0 1729 ( n 1 ) ( 1728 n 1 ) ( 1 2 ) 1729 + n = 0 1729 ( 1728 n 1 ) ( 1 2 ) 1729 ] = 1729 [ 1728 n = 0 1729 ( 1727 n 2 ) ( 1 2 ) 1729 + n = 1 1728 ( 1728 n ) ( 1 2 ) 1729 ] = 1729 [ 1728 n = 2 1727 ( 1727 n ) ( 1 2 ) 1729 + 1 2 n = 0 1728 ( 1728 n ) ( 1 2 ) n ( 1 2 ) 1728 n ] = 1729 [ 1728 4 n = 0 1727 ( 1727 n ) ( 1 2 ) n ( 1 2 ) 1727 n + 1 2 ] = 1729 [ 1728 4 + 1 2 ] = 1730 × 1729 4 \\ \textbf{Manipulating } S_1 \\ \sum_{n=0}^{1729}n^2{1729 \choose n}(\frac{1}{2})^{1729}\\ =1729\sum_{n=0}^{1729}(n-1+1){1728 \choose {n-1}}(\frac{1}{2})^{1729}\\ =1729\Big[\sum_{n=0}^{1729}(n-1){1728 \choose {n-1}}(\frac{1}{2})^{1729}+\sum_{n=0}^{1729}{1728 \choose {n-1}}(\frac{1}{2})^{1729}\Big]\\ =1729\Big[1728\sum_{n=0}^{1729}{1727 \choose {n-2}}(\frac{1}{2})^{1729}+\sum_{n=-1}^{1728}{1728 \choose {n}}(\frac{1}{2})^{1729}\Big]\\ =1729\Big[1728\sum_{n=-2}^{1727}{1727 \choose n}(\frac{1}{2})^{1729}+\frac{1}{2}\sum_{n=0}^{1728}{1728 \choose {n}}(\frac{1}{2})^{n}(\frac{1}{2})^{1728-n}\Big]\\ =1729\Big[\frac{1728}{4}\sum_{n=0}^{1727}{1727 \choose n}(\frac{1}{2})^{n}(\frac{1}{2})^{1727-n}+\frac{1}{2}\Big]\\ =1729\Big[\frac{1728}{4}+\frac{1}{2}\Big]=\frac{1730 \times 1729}{4}

Now, for S S = 1730 × 1729 4 1729 × 1729 2 + 172 9 2 4 = 1729 4 So, the answer is $1729+4= 1733 $ \\ \textbf{Now, for } S \\ S=\frac{1730 \times 1729}{4}-\frac{1729 \times 1729}{2}+\frac{1729^2}{4}=\boxed{\frac{1729}{4}} \\ \text{So, the answer is \$1729+4=\boxed{1733}\$}

Nice one! Thanks for posting the solution :)

Aditya Kumar - 5 years, 3 months ago

0 pending reports

×

Problem Loading...

Note Loading...

Set Loading...